Automorphism group of the quaternion group












13












$begingroup$


Let $Q_8$ be the quaternion group.
How do we determine the automorphism group $Aut(Q_8)$ of $Q_8$ algebraically?
I searched for this problem on internet.
I found some geometric proofs that $Aut(Q_8)$ is isomorphic to the rotation group of a cube, hence it is isomorphic to the symmetric group $S_4$.
I would like to know an algebraic proof that $Aut(Q_8)$ is isomorphic to $S_4$.










share|cite|improve this question











$endgroup$












  • $begingroup$
    Have a look at this: crazyproject.wordpress.com/2010/07/25/…. It basically boils down to the fact that no automorphism of $Q_8$ can have order 6.
    $endgroup$
    – M Turgeon
    Sep 14 '12 at 23:17












  • $begingroup$
    @MTurgeon Thanks. "We already know that $|Aut(Q_8)| = 24$ by counting the possible images of, say. $i$ and $j$". How do we know that?
    $endgroup$
    – Makoto Kato
    Sep 14 '12 at 23:23






  • 2




    $begingroup$
    $i$ can go to any of the $6$ elements of order $4$ in $Q_8$. After that, $j$ can go to any of four elements of order $4$; it can't end up in the same subgroup as $i$ (it won't be an automorphism), and since $i$ and $j$ generate $Q_8$, this is sufficient to determine the automorphism.
    $endgroup$
    – user641
    Sep 14 '12 at 23:34






  • 2




    $begingroup$
    Now simply note that any permutation of $i,j,k$ gives an autmorphism. Each of these is outer, since the subgroups $langle irangle$, $langle jrangle$, and $langle krangle$ are normal. Thus the automorphism group contains the semidirect product of the inner automorphisms (of size 4), and the permutations on $i,j,k$ (of size 6). Now you are done.
    $endgroup$
    – user641
    Sep 14 '12 at 23:38










  • $begingroup$
    @SteveD Why don't you make it the answer?
    $endgroup$
    – Makoto Kato
    Sep 15 '12 at 2:22
















13












$begingroup$


Let $Q_8$ be the quaternion group.
How do we determine the automorphism group $Aut(Q_8)$ of $Q_8$ algebraically?
I searched for this problem on internet.
I found some geometric proofs that $Aut(Q_8)$ is isomorphic to the rotation group of a cube, hence it is isomorphic to the symmetric group $S_4$.
I would like to know an algebraic proof that $Aut(Q_8)$ is isomorphic to $S_4$.










share|cite|improve this question











$endgroup$












  • $begingroup$
    Have a look at this: crazyproject.wordpress.com/2010/07/25/…. It basically boils down to the fact that no automorphism of $Q_8$ can have order 6.
    $endgroup$
    – M Turgeon
    Sep 14 '12 at 23:17












  • $begingroup$
    @MTurgeon Thanks. "We already know that $|Aut(Q_8)| = 24$ by counting the possible images of, say. $i$ and $j$". How do we know that?
    $endgroup$
    – Makoto Kato
    Sep 14 '12 at 23:23






  • 2




    $begingroup$
    $i$ can go to any of the $6$ elements of order $4$ in $Q_8$. After that, $j$ can go to any of four elements of order $4$; it can't end up in the same subgroup as $i$ (it won't be an automorphism), and since $i$ and $j$ generate $Q_8$, this is sufficient to determine the automorphism.
    $endgroup$
    – user641
    Sep 14 '12 at 23:34






  • 2




    $begingroup$
    Now simply note that any permutation of $i,j,k$ gives an autmorphism. Each of these is outer, since the subgroups $langle irangle$, $langle jrangle$, and $langle krangle$ are normal. Thus the automorphism group contains the semidirect product of the inner automorphisms (of size 4), and the permutations on $i,j,k$ (of size 6). Now you are done.
    $endgroup$
    – user641
    Sep 14 '12 at 23:38










  • $begingroup$
    @SteveD Why don't you make it the answer?
    $endgroup$
    – Makoto Kato
    Sep 15 '12 at 2:22














13












13








13


12



$begingroup$


Let $Q_8$ be the quaternion group.
How do we determine the automorphism group $Aut(Q_8)$ of $Q_8$ algebraically?
I searched for this problem on internet.
I found some geometric proofs that $Aut(Q_8)$ is isomorphic to the rotation group of a cube, hence it is isomorphic to the symmetric group $S_4$.
I would like to know an algebraic proof that $Aut(Q_8)$ is isomorphic to $S_4$.










share|cite|improve this question











$endgroup$




Let $Q_8$ be the quaternion group.
How do we determine the automorphism group $Aut(Q_8)$ of $Q_8$ algebraically?
I searched for this problem on internet.
I found some geometric proofs that $Aut(Q_8)$ is isomorphic to the rotation group of a cube, hence it is isomorphic to the symmetric group $S_4$.
I would like to know an algebraic proof that $Aut(Q_8)$ is isomorphic to $S_4$.







group-theory finite-groups






share|cite|improve this question















share|cite|improve this question













share|cite|improve this question




share|cite|improve this question








edited Sep 14 '12 at 23:13









M Turgeon

7,87133066




7,87133066










asked Sep 14 '12 at 23:05









Makoto KatoMakoto Kato

23k560162




23k560162












  • $begingroup$
    Have a look at this: crazyproject.wordpress.com/2010/07/25/…. It basically boils down to the fact that no automorphism of $Q_8$ can have order 6.
    $endgroup$
    – M Turgeon
    Sep 14 '12 at 23:17












  • $begingroup$
    @MTurgeon Thanks. "We already know that $|Aut(Q_8)| = 24$ by counting the possible images of, say. $i$ and $j$". How do we know that?
    $endgroup$
    – Makoto Kato
    Sep 14 '12 at 23:23






  • 2




    $begingroup$
    $i$ can go to any of the $6$ elements of order $4$ in $Q_8$. After that, $j$ can go to any of four elements of order $4$; it can't end up in the same subgroup as $i$ (it won't be an automorphism), and since $i$ and $j$ generate $Q_8$, this is sufficient to determine the automorphism.
    $endgroup$
    – user641
    Sep 14 '12 at 23:34






  • 2




    $begingroup$
    Now simply note that any permutation of $i,j,k$ gives an autmorphism. Each of these is outer, since the subgroups $langle irangle$, $langle jrangle$, and $langle krangle$ are normal. Thus the automorphism group contains the semidirect product of the inner automorphisms (of size 4), and the permutations on $i,j,k$ (of size 6). Now you are done.
    $endgroup$
    – user641
    Sep 14 '12 at 23:38










  • $begingroup$
    @SteveD Why don't you make it the answer?
    $endgroup$
    – Makoto Kato
    Sep 15 '12 at 2:22


















  • $begingroup$
    Have a look at this: crazyproject.wordpress.com/2010/07/25/…. It basically boils down to the fact that no automorphism of $Q_8$ can have order 6.
    $endgroup$
    – M Turgeon
    Sep 14 '12 at 23:17












  • $begingroup$
    @MTurgeon Thanks. "We already know that $|Aut(Q_8)| = 24$ by counting the possible images of, say. $i$ and $j$". How do we know that?
    $endgroup$
    – Makoto Kato
    Sep 14 '12 at 23:23






  • 2




    $begingroup$
    $i$ can go to any of the $6$ elements of order $4$ in $Q_8$. After that, $j$ can go to any of four elements of order $4$; it can't end up in the same subgroup as $i$ (it won't be an automorphism), and since $i$ and $j$ generate $Q_8$, this is sufficient to determine the automorphism.
    $endgroup$
    – user641
    Sep 14 '12 at 23:34






  • 2




    $begingroup$
    Now simply note that any permutation of $i,j,k$ gives an autmorphism. Each of these is outer, since the subgroups $langle irangle$, $langle jrangle$, and $langle krangle$ are normal. Thus the automorphism group contains the semidirect product of the inner automorphisms (of size 4), and the permutations on $i,j,k$ (of size 6). Now you are done.
    $endgroup$
    – user641
    Sep 14 '12 at 23:38










  • $begingroup$
    @SteveD Why don't you make it the answer?
    $endgroup$
    – Makoto Kato
    Sep 15 '12 at 2:22
















$begingroup$
Have a look at this: crazyproject.wordpress.com/2010/07/25/…. It basically boils down to the fact that no automorphism of $Q_8$ can have order 6.
$endgroup$
– M Turgeon
Sep 14 '12 at 23:17






$begingroup$
Have a look at this: crazyproject.wordpress.com/2010/07/25/…. It basically boils down to the fact that no automorphism of $Q_8$ can have order 6.
$endgroup$
– M Turgeon
Sep 14 '12 at 23:17














$begingroup$
@MTurgeon Thanks. "We already know that $|Aut(Q_8)| = 24$ by counting the possible images of, say. $i$ and $j$". How do we know that?
$endgroup$
– Makoto Kato
Sep 14 '12 at 23:23




$begingroup$
@MTurgeon Thanks. "We already know that $|Aut(Q_8)| = 24$ by counting the possible images of, say. $i$ and $j$". How do we know that?
$endgroup$
– Makoto Kato
Sep 14 '12 at 23:23




2




2




$begingroup$
$i$ can go to any of the $6$ elements of order $4$ in $Q_8$. After that, $j$ can go to any of four elements of order $4$; it can't end up in the same subgroup as $i$ (it won't be an automorphism), and since $i$ and $j$ generate $Q_8$, this is sufficient to determine the automorphism.
$endgroup$
– user641
Sep 14 '12 at 23:34




$begingroup$
$i$ can go to any of the $6$ elements of order $4$ in $Q_8$. After that, $j$ can go to any of four elements of order $4$; it can't end up in the same subgroup as $i$ (it won't be an automorphism), and since $i$ and $j$ generate $Q_8$, this is sufficient to determine the automorphism.
$endgroup$
– user641
Sep 14 '12 at 23:34




2




2




$begingroup$
Now simply note that any permutation of $i,j,k$ gives an autmorphism. Each of these is outer, since the subgroups $langle irangle$, $langle jrangle$, and $langle krangle$ are normal. Thus the automorphism group contains the semidirect product of the inner automorphisms (of size 4), and the permutations on $i,j,k$ (of size 6). Now you are done.
$endgroup$
– user641
Sep 14 '12 at 23:38




$begingroup$
Now simply note that any permutation of $i,j,k$ gives an autmorphism. Each of these is outer, since the subgroups $langle irangle$, $langle jrangle$, and $langle krangle$ are normal. Thus the automorphism group contains the semidirect product of the inner automorphisms (of size 4), and the permutations on $i,j,k$ (of size 6). Now you are done.
$endgroup$
– user641
Sep 14 '12 at 23:38












$begingroup$
@SteveD Why don't you make it the answer?
$endgroup$
– Makoto Kato
Sep 15 '12 at 2:22




$begingroup$
@SteveD Why don't you make it the answer?
$endgroup$
– Makoto Kato
Sep 15 '12 at 2:22










5 Answers
5






active

oldest

votes


















9












$begingroup$

$Q_8$ has three cyclic subgroups of order 4: $langle irangle$, $langle jrangle$, $langle krangle$, and $Aut(Q_8)$ acts on these three subgroups; inducing a homomorphism $Phicolon Aut(Q_8)rightarrow S_3$. We can see that, the homomorphism is surjective, since the two automorphisms $fcolon imapsto j, jmapsto i$, and $gcolon jmapsto k, kmapsto j$ give two transpositions in $S_3$.
The kernel contains those $varphiin Aut(G)$ such that $varphi(langle irangle)=langle irangle$ and $varphi(langle jrangle)=langle jrangle$ (automatically, $varphi(langle krangle)=langle krangle$).



(1) $varphi(langle irangle)=langle irangle$ means $varphi(i)in {i,-i}$, and similarly, $varphi(j)in {j,-j}$. One can check that these four choices are automorphisms of order 2 (or 1) (since they are switching elements in a pair), and hence kernel is Klein-4 group $V_4$.



(2) Since, automorphisms in the kernel fix all the cyclic subgroups of $Q_8$ (not necessarily point-wise); consider the following automorphisms:



$S: imapsto j, jmapsto i$, (hence $kmapsto -k$) and



$Tcolon jmapsto k, kmapsto j$ (hence $imapsto -i$);



these are not inner (since they fix a subgroup), and they generate $S_3$ (they are like transpositions $(1,2)$ and $(2,3)$ ). Therefore, we have $langle S,T rangle=Kleq Aut(Q_8)$, such that $Kcong S_3$ and $Phi(K)=S_3$. Also,




$ker(Phi) cap K=phi $.




Therefore, $Aut(Q_8)=ker(Phi)rtimes K cong V_4rtimes S_3$.



Consider an element of $ker(Phi)$:



$f:imapsto -i$, $jmapsto j$,



and two elements of $Kcong Im$:



$gcolon imapsto j, jmapsto i $ (like a transposition), and $hcolon imapsto j, jmapsto k$ (like a 3-cycle).



One can check that $f$ doesn't commute with $g$ as well as $h$.



In fact, this shows that no element of $V_4setminus{1}$ commutes with any element of $Ksetminus { 1}$ (by inter-changing roles of $i,j,k$); this means, the action of $K$ on $V_4$ (by conjugation) is faithful; and up to equivalence, there is only one such action. Therefore, $Aut(Q_8)=V_4rtimes Kcong S_4$






share|cite|improve this answer











$endgroup$













  • $begingroup$
    Could you explain the step (2)? I don't understand why we have a subgroup $K$ isomorphic to $S_3$.
    $endgroup$
    – Makoto Kato
    Sep 15 '12 at 14:13










  • $begingroup$
    @Kato: nice question; edited the answer.
    $endgroup$
    – Beginner
    Sep 15 '12 at 15:23



















5












$begingroup$

OK, let's first put an upper bound on the number of automorphisms of $Q_8$.



There are $6$ elements of order $4$ in $Q_8$. It is well-known that $i$ and $j$ generate $Q_8$, so their images under some automorphism $phi$ are enough to determine that automorphism uniquely. So there are $6$ choices for $phi(i)$. Now we cannot have $phi(j)inlanglephi(i)rangle$, because then $phi$ would fail to be surjective. Thus there are $6-2=4$ possibilities for $phi(j)$. This crude reasoning gives the upper bound of $6cdot4=24$ automorphisms.



Let $alpha$ be any permutation on the elements $lbrace i,j,krbrace$ (as a set). We can "extend" $alpha$ to a map of $Q_8$ in the obvious way (let it commute with negatives). Since any two of $lbrace i,j,krbrace$ generates $Q_8$, this map will be surjective, and since $Q_8$ is finite, injective as well. It only remains to show it is a homomorphism. Since $i^2=j^2=k^2=-1$ is central and the only element of order $2$, we only need show
$$ alpha(i)alpha(j)=alpha(k);$$



This is not always true, but what is always true is that
$$ alpha(i)alpha(j)=pmalpha(k),$$
and so we see each permutation gives rise to an automorphism (after a suitable choice of sign).



Since the subgroups $langle irangle$,$langle jrangle$,$langle krangle$ are normal, none of these permutations is an inner automorphism. Thus they give a subgroup isomorphic to $S_3$ inside $Aut(Q_8)$, which trivially intersects $Inn(Q_8)cong Q_8/Z(Q_8)cong C_2times C_2$. Thus $Aut(Q_8)$ has size at least $|S_3|cdot |C_2times C_2|=24$ automorphisms.



Since $Inn(Q_8)lhd Aut(Q_8)$, we get a semidirect product $(C_2times C_2)rtimes S_3$ inside $Aut(Q_8)$, which is then the whole group $Aut(Q_8)$. This is easily seen to be isomorphic to $S_4$, and so we are done.






share|cite|improve this answer









$endgroup$













  • $begingroup$
    "Since $i^2=j^2=k^2=-1$ is central and the only element of order $2$, we only need show $ alpha(i)alpha(j)=alpha(k);$ This is not always true, but what is always true is that $ alpha(i)alpha(j)=pmalpha(k),$ and so we see each permutation gives rise to an automorphism (after a suitable choice of sign)." I think the explanations are a bit too terse. Could you explain in more detail?
    $endgroup$
    – Makoto Kato
    Sep 15 '12 at 13:55



















3












$begingroup$

Hint:



$Inn(Q_8)cong V$ and it equals its own centralizer in $Aut(Q_8)$. Now use $N/C$ Lemma in which $G=Aut(Q_8)$ and $H=Inn(Q_8)$. Of course using the lemma we always have $Inn(G)vartriangleleft Aut(G)$ and $G/Z(G)cong Inn(G)$ in which $G$ is our group.






share|cite|improve this answer









$endgroup$













  • $begingroup$
    What is $N/C$ Lemma?
    $endgroup$
    – Makoto Kato
    Sep 15 '12 at 13:58






  • 1




    $begingroup$
    If $Hleq G$, then $C_{G}(H)vartriangleleft N_{G}(H)$ and moreover $N_{G}(H)/C_{G}(H)hookrightarrow Aut(H)$. :)
    $endgroup$
    – mrs
    Sep 15 '12 at 14:50








  • 1




    $begingroup$
    math.stackexchange.com/a/30379/583 has the proof for Inn being its own centralizer in this case.
    $endgroup$
    – Jack Schmidt
    Sep 15 '12 at 18:45










  • $begingroup$
    $quad +1quad ddotsmilequad$
    $endgroup$
    – Namaste
    Mar 18 '13 at 0:56



















1












$begingroup$

You can find the proof here: automorphism of generalized quaternionic group



See Proposition 1.1 (pg. 156). Your case can be obtained taking $m=2$. But they prove for $mgeq 2$.






share|cite|improve this answer











$endgroup$





















    0












    $begingroup$

    The simplest way to determine the automorphism group of $Q_{8}$ may be to realize that $$Q_{8} text{char} SL(2,3) unlhd GL(2,3)$$ So $Q_{8} unlhd GL(2,3)$. One can take $$begin{equation*} x =left( begin{array}{cc} 0 & 1 \ -1 & 0 end{array} right), y = left( begin{array}{cc} 1 & -1 \ -1 & -1 end{array} right) end{equation*}$$ as the two generators of $Q_{8}$.



    Let $G=GL(2,3)$ and $Q=Q_{8}$. Then $C_{G}(Q) = Z(G)$ and therefore $PGL(2,3)=G/Z(G)$ is isomorphic to a subgroup of $Aut(Q)$. Now $|G|=48$ and $|Z(G)|=2$. Thus $|G/Z(G)|=24$. As others have shown $|Aut(Q)| leq 24$. Hence we can conclude that $$PGL(2,3) simeq Aut(Q_{8})$$
    Using that $SL(2,3)$ and therefore $GL(2,3)$ have $4$ Sylow-$3$ subgroups one get a homomorphism into $Sym(4)$. It is straight forward to show that the kernel of this homomorphism is $Z(GL(2,3))$ and therefore $PGL(2,3)$ is isomorphic to a subgroup of $Sym(4)$. Since the two groups have the same number of elements we are done.






    share|cite|improve this answer









    $endgroup$













      Your Answer





      StackExchange.ifUsing("editor", function () {
      return StackExchange.using("mathjaxEditing", function () {
      StackExchange.MarkdownEditor.creationCallbacks.add(function (editor, postfix) {
      StackExchange.mathjaxEditing.prepareWmdForMathJax(editor, postfix, [["$", "$"], ["\\(","\\)"]]);
      });
      });
      }, "mathjax-editing");

      StackExchange.ready(function() {
      var channelOptions = {
      tags: "".split(" "),
      id: "69"
      };
      initTagRenderer("".split(" "), "".split(" "), channelOptions);

      StackExchange.using("externalEditor", function() {
      // Have to fire editor after snippets, if snippets enabled
      if (StackExchange.settings.snippets.snippetsEnabled) {
      StackExchange.using("snippets", function() {
      createEditor();
      });
      }
      else {
      createEditor();
      }
      });

      function createEditor() {
      StackExchange.prepareEditor({
      heartbeatType: 'answer',
      autoActivateHeartbeat: false,
      convertImagesToLinks: true,
      noModals: true,
      showLowRepImageUploadWarning: true,
      reputationToPostImages: 10,
      bindNavPrevention: true,
      postfix: "",
      imageUploader: {
      brandingHtml: "Powered by u003ca class="icon-imgur-white" href="https://imgur.com/"u003eu003c/au003e",
      contentPolicyHtml: "User contributions licensed under u003ca href="https://creativecommons.org/licenses/by-sa/3.0/"u003ecc by-sa 3.0 with attribution requiredu003c/au003e u003ca href="https://stackoverflow.com/legal/content-policy"u003e(content policy)u003c/au003e",
      allowUrls: true
      },
      noCode: true, onDemand: true,
      discardSelector: ".discard-answer"
      ,immediatelyShowMarkdownHelp:true
      });


      }
      });














      draft saved

      draft discarded


















      StackExchange.ready(
      function () {
      StackExchange.openid.initPostLogin('.new-post-login', 'https%3a%2f%2fmath.stackexchange.com%2fquestions%2f195932%2fautomorphism-group-of-the-quaternion-group%23new-answer', 'question_page');
      }
      );

      Post as a guest















      Required, but never shown

























      5 Answers
      5






      active

      oldest

      votes








      5 Answers
      5






      active

      oldest

      votes









      active

      oldest

      votes






      active

      oldest

      votes









      9












      $begingroup$

      $Q_8$ has three cyclic subgroups of order 4: $langle irangle$, $langle jrangle$, $langle krangle$, and $Aut(Q_8)$ acts on these three subgroups; inducing a homomorphism $Phicolon Aut(Q_8)rightarrow S_3$. We can see that, the homomorphism is surjective, since the two automorphisms $fcolon imapsto j, jmapsto i$, and $gcolon jmapsto k, kmapsto j$ give two transpositions in $S_3$.
      The kernel contains those $varphiin Aut(G)$ such that $varphi(langle irangle)=langle irangle$ and $varphi(langle jrangle)=langle jrangle$ (automatically, $varphi(langle krangle)=langle krangle$).



      (1) $varphi(langle irangle)=langle irangle$ means $varphi(i)in {i,-i}$, and similarly, $varphi(j)in {j,-j}$. One can check that these four choices are automorphisms of order 2 (or 1) (since they are switching elements in a pair), and hence kernel is Klein-4 group $V_4$.



      (2) Since, automorphisms in the kernel fix all the cyclic subgroups of $Q_8$ (not necessarily point-wise); consider the following automorphisms:



      $S: imapsto j, jmapsto i$, (hence $kmapsto -k$) and



      $Tcolon jmapsto k, kmapsto j$ (hence $imapsto -i$);



      these are not inner (since they fix a subgroup), and they generate $S_3$ (they are like transpositions $(1,2)$ and $(2,3)$ ). Therefore, we have $langle S,T rangle=Kleq Aut(Q_8)$, such that $Kcong S_3$ and $Phi(K)=S_3$. Also,




      $ker(Phi) cap K=phi $.




      Therefore, $Aut(Q_8)=ker(Phi)rtimes K cong V_4rtimes S_3$.



      Consider an element of $ker(Phi)$:



      $f:imapsto -i$, $jmapsto j$,



      and two elements of $Kcong Im$:



      $gcolon imapsto j, jmapsto i $ (like a transposition), and $hcolon imapsto j, jmapsto k$ (like a 3-cycle).



      One can check that $f$ doesn't commute with $g$ as well as $h$.



      In fact, this shows that no element of $V_4setminus{1}$ commutes with any element of $Ksetminus { 1}$ (by inter-changing roles of $i,j,k$); this means, the action of $K$ on $V_4$ (by conjugation) is faithful; and up to equivalence, there is only one such action. Therefore, $Aut(Q_8)=V_4rtimes Kcong S_4$






      share|cite|improve this answer











      $endgroup$













      • $begingroup$
        Could you explain the step (2)? I don't understand why we have a subgroup $K$ isomorphic to $S_3$.
        $endgroup$
        – Makoto Kato
        Sep 15 '12 at 14:13










      • $begingroup$
        @Kato: nice question; edited the answer.
        $endgroup$
        – Beginner
        Sep 15 '12 at 15:23
















      9












      $begingroup$

      $Q_8$ has three cyclic subgroups of order 4: $langle irangle$, $langle jrangle$, $langle krangle$, and $Aut(Q_8)$ acts on these three subgroups; inducing a homomorphism $Phicolon Aut(Q_8)rightarrow S_3$. We can see that, the homomorphism is surjective, since the two automorphisms $fcolon imapsto j, jmapsto i$, and $gcolon jmapsto k, kmapsto j$ give two transpositions in $S_3$.
      The kernel contains those $varphiin Aut(G)$ such that $varphi(langle irangle)=langle irangle$ and $varphi(langle jrangle)=langle jrangle$ (automatically, $varphi(langle krangle)=langle krangle$).



      (1) $varphi(langle irangle)=langle irangle$ means $varphi(i)in {i,-i}$, and similarly, $varphi(j)in {j,-j}$. One can check that these four choices are automorphisms of order 2 (or 1) (since they are switching elements in a pair), and hence kernel is Klein-4 group $V_4$.



      (2) Since, automorphisms in the kernel fix all the cyclic subgroups of $Q_8$ (not necessarily point-wise); consider the following automorphisms:



      $S: imapsto j, jmapsto i$, (hence $kmapsto -k$) and



      $Tcolon jmapsto k, kmapsto j$ (hence $imapsto -i$);



      these are not inner (since they fix a subgroup), and they generate $S_3$ (they are like transpositions $(1,2)$ and $(2,3)$ ). Therefore, we have $langle S,T rangle=Kleq Aut(Q_8)$, such that $Kcong S_3$ and $Phi(K)=S_3$. Also,




      $ker(Phi) cap K=phi $.




      Therefore, $Aut(Q_8)=ker(Phi)rtimes K cong V_4rtimes S_3$.



      Consider an element of $ker(Phi)$:



      $f:imapsto -i$, $jmapsto j$,



      and two elements of $Kcong Im$:



      $gcolon imapsto j, jmapsto i $ (like a transposition), and $hcolon imapsto j, jmapsto k$ (like a 3-cycle).



      One can check that $f$ doesn't commute with $g$ as well as $h$.



      In fact, this shows that no element of $V_4setminus{1}$ commutes with any element of $Ksetminus { 1}$ (by inter-changing roles of $i,j,k$); this means, the action of $K$ on $V_4$ (by conjugation) is faithful; and up to equivalence, there is only one such action. Therefore, $Aut(Q_8)=V_4rtimes Kcong S_4$






      share|cite|improve this answer











      $endgroup$













      • $begingroup$
        Could you explain the step (2)? I don't understand why we have a subgroup $K$ isomorphic to $S_3$.
        $endgroup$
        – Makoto Kato
        Sep 15 '12 at 14:13










      • $begingroup$
        @Kato: nice question; edited the answer.
        $endgroup$
        – Beginner
        Sep 15 '12 at 15:23














      9












      9








      9





      $begingroup$

      $Q_8$ has three cyclic subgroups of order 4: $langle irangle$, $langle jrangle$, $langle krangle$, and $Aut(Q_8)$ acts on these three subgroups; inducing a homomorphism $Phicolon Aut(Q_8)rightarrow S_3$. We can see that, the homomorphism is surjective, since the two automorphisms $fcolon imapsto j, jmapsto i$, and $gcolon jmapsto k, kmapsto j$ give two transpositions in $S_3$.
      The kernel contains those $varphiin Aut(G)$ such that $varphi(langle irangle)=langle irangle$ and $varphi(langle jrangle)=langle jrangle$ (automatically, $varphi(langle krangle)=langle krangle$).



      (1) $varphi(langle irangle)=langle irangle$ means $varphi(i)in {i,-i}$, and similarly, $varphi(j)in {j,-j}$. One can check that these four choices are automorphisms of order 2 (or 1) (since they are switching elements in a pair), and hence kernel is Klein-4 group $V_4$.



      (2) Since, automorphisms in the kernel fix all the cyclic subgroups of $Q_8$ (not necessarily point-wise); consider the following automorphisms:



      $S: imapsto j, jmapsto i$, (hence $kmapsto -k$) and



      $Tcolon jmapsto k, kmapsto j$ (hence $imapsto -i$);



      these are not inner (since they fix a subgroup), and they generate $S_3$ (they are like transpositions $(1,2)$ and $(2,3)$ ). Therefore, we have $langle S,T rangle=Kleq Aut(Q_8)$, such that $Kcong S_3$ and $Phi(K)=S_3$. Also,




      $ker(Phi) cap K=phi $.




      Therefore, $Aut(Q_8)=ker(Phi)rtimes K cong V_4rtimes S_3$.



      Consider an element of $ker(Phi)$:



      $f:imapsto -i$, $jmapsto j$,



      and two elements of $Kcong Im$:



      $gcolon imapsto j, jmapsto i $ (like a transposition), and $hcolon imapsto j, jmapsto k$ (like a 3-cycle).



      One can check that $f$ doesn't commute with $g$ as well as $h$.



      In fact, this shows that no element of $V_4setminus{1}$ commutes with any element of $Ksetminus { 1}$ (by inter-changing roles of $i,j,k$); this means, the action of $K$ on $V_4$ (by conjugation) is faithful; and up to equivalence, there is only one such action. Therefore, $Aut(Q_8)=V_4rtimes Kcong S_4$






      share|cite|improve this answer











      $endgroup$



      $Q_8$ has three cyclic subgroups of order 4: $langle irangle$, $langle jrangle$, $langle krangle$, and $Aut(Q_8)$ acts on these three subgroups; inducing a homomorphism $Phicolon Aut(Q_8)rightarrow S_3$. We can see that, the homomorphism is surjective, since the two automorphisms $fcolon imapsto j, jmapsto i$, and $gcolon jmapsto k, kmapsto j$ give two transpositions in $S_3$.
      The kernel contains those $varphiin Aut(G)$ such that $varphi(langle irangle)=langle irangle$ and $varphi(langle jrangle)=langle jrangle$ (automatically, $varphi(langle krangle)=langle krangle$).



      (1) $varphi(langle irangle)=langle irangle$ means $varphi(i)in {i,-i}$, and similarly, $varphi(j)in {j,-j}$. One can check that these four choices are automorphisms of order 2 (or 1) (since they are switching elements in a pair), and hence kernel is Klein-4 group $V_4$.



      (2) Since, automorphisms in the kernel fix all the cyclic subgroups of $Q_8$ (not necessarily point-wise); consider the following automorphisms:



      $S: imapsto j, jmapsto i$, (hence $kmapsto -k$) and



      $Tcolon jmapsto k, kmapsto j$ (hence $imapsto -i$);



      these are not inner (since they fix a subgroup), and they generate $S_3$ (they are like transpositions $(1,2)$ and $(2,3)$ ). Therefore, we have $langle S,T rangle=Kleq Aut(Q_8)$, such that $Kcong S_3$ and $Phi(K)=S_3$. Also,




      $ker(Phi) cap K=phi $.




      Therefore, $Aut(Q_8)=ker(Phi)rtimes K cong V_4rtimes S_3$.



      Consider an element of $ker(Phi)$:



      $f:imapsto -i$, $jmapsto j$,



      and two elements of $Kcong Im$:



      $gcolon imapsto j, jmapsto i $ (like a transposition), and $hcolon imapsto j, jmapsto k$ (like a 3-cycle).



      One can check that $f$ doesn't commute with $g$ as well as $h$.



      In fact, this shows that no element of $V_4setminus{1}$ commutes with any element of $Ksetminus { 1}$ (by inter-changing roles of $i,j,k$); this means, the action of $K$ on $V_4$ (by conjugation) is faithful; and up to equivalence, there is only one such action. Therefore, $Aut(Q_8)=V_4rtimes Kcong S_4$







      share|cite|improve this answer














      share|cite|improve this answer



      share|cite|improve this answer








      edited Sep 15 '12 at 15:21

























      answered Sep 15 '12 at 5:37









      BeginnerBeginner

      3,97611226




      3,97611226












      • $begingroup$
        Could you explain the step (2)? I don't understand why we have a subgroup $K$ isomorphic to $S_3$.
        $endgroup$
        – Makoto Kato
        Sep 15 '12 at 14:13










      • $begingroup$
        @Kato: nice question; edited the answer.
        $endgroup$
        – Beginner
        Sep 15 '12 at 15:23


















      • $begingroup$
        Could you explain the step (2)? I don't understand why we have a subgroup $K$ isomorphic to $S_3$.
        $endgroup$
        – Makoto Kato
        Sep 15 '12 at 14:13










      • $begingroup$
        @Kato: nice question; edited the answer.
        $endgroup$
        – Beginner
        Sep 15 '12 at 15:23
















      $begingroup$
      Could you explain the step (2)? I don't understand why we have a subgroup $K$ isomorphic to $S_3$.
      $endgroup$
      – Makoto Kato
      Sep 15 '12 at 14:13




      $begingroup$
      Could you explain the step (2)? I don't understand why we have a subgroup $K$ isomorphic to $S_3$.
      $endgroup$
      – Makoto Kato
      Sep 15 '12 at 14:13












      $begingroup$
      @Kato: nice question; edited the answer.
      $endgroup$
      – Beginner
      Sep 15 '12 at 15:23




      $begingroup$
      @Kato: nice question; edited the answer.
      $endgroup$
      – Beginner
      Sep 15 '12 at 15:23











      5












      $begingroup$

      OK, let's first put an upper bound on the number of automorphisms of $Q_8$.



      There are $6$ elements of order $4$ in $Q_8$. It is well-known that $i$ and $j$ generate $Q_8$, so their images under some automorphism $phi$ are enough to determine that automorphism uniquely. So there are $6$ choices for $phi(i)$. Now we cannot have $phi(j)inlanglephi(i)rangle$, because then $phi$ would fail to be surjective. Thus there are $6-2=4$ possibilities for $phi(j)$. This crude reasoning gives the upper bound of $6cdot4=24$ automorphisms.



      Let $alpha$ be any permutation on the elements $lbrace i,j,krbrace$ (as a set). We can "extend" $alpha$ to a map of $Q_8$ in the obvious way (let it commute with negatives). Since any two of $lbrace i,j,krbrace$ generates $Q_8$, this map will be surjective, and since $Q_8$ is finite, injective as well. It only remains to show it is a homomorphism. Since $i^2=j^2=k^2=-1$ is central and the only element of order $2$, we only need show
      $$ alpha(i)alpha(j)=alpha(k);$$



      This is not always true, but what is always true is that
      $$ alpha(i)alpha(j)=pmalpha(k),$$
      and so we see each permutation gives rise to an automorphism (after a suitable choice of sign).



      Since the subgroups $langle irangle$,$langle jrangle$,$langle krangle$ are normal, none of these permutations is an inner automorphism. Thus they give a subgroup isomorphic to $S_3$ inside $Aut(Q_8)$, which trivially intersects $Inn(Q_8)cong Q_8/Z(Q_8)cong C_2times C_2$. Thus $Aut(Q_8)$ has size at least $|S_3|cdot |C_2times C_2|=24$ automorphisms.



      Since $Inn(Q_8)lhd Aut(Q_8)$, we get a semidirect product $(C_2times C_2)rtimes S_3$ inside $Aut(Q_8)$, which is then the whole group $Aut(Q_8)$. This is easily seen to be isomorphic to $S_4$, and so we are done.






      share|cite|improve this answer









      $endgroup$













      • $begingroup$
        "Since $i^2=j^2=k^2=-1$ is central and the only element of order $2$, we only need show $ alpha(i)alpha(j)=alpha(k);$ This is not always true, but what is always true is that $ alpha(i)alpha(j)=pmalpha(k),$ and so we see each permutation gives rise to an automorphism (after a suitable choice of sign)." I think the explanations are a bit too terse. Could you explain in more detail?
        $endgroup$
        – Makoto Kato
        Sep 15 '12 at 13:55
















      5












      $begingroup$

      OK, let's first put an upper bound on the number of automorphisms of $Q_8$.



      There are $6$ elements of order $4$ in $Q_8$. It is well-known that $i$ and $j$ generate $Q_8$, so their images under some automorphism $phi$ are enough to determine that automorphism uniquely. So there are $6$ choices for $phi(i)$. Now we cannot have $phi(j)inlanglephi(i)rangle$, because then $phi$ would fail to be surjective. Thus there are $6-2=4$ possibilities for $phi(j)$. This crude reasoning gives the upper bound of $6cdot4=24$ automorphisms.



      Let $alpha$ be any permutation on the elements $lbrace i,j,krbrace$ (as a set). We can "extend" $alpha$ to a map of $Q_8$ in the obvious way (let it commute with negatives). Since any two of $lbrace i,j,krbrace$ generates $Q_8$, this map will be surjective, and since $Q_8$ is finite, injective as well. It only remains to show it is a homomorphism. Since $i^2=j^2=k^2=-1$ is central and the only element of order $2$, we only need show
      $$ alpha(i)alpha(j)=alpha(k);$$



      This is not always true, but what is always true is that
      $$ alpha(i)alpha(j)=pmalpha(k),$$
      and so we see each permutation gives rise to an automorphism (after a suitable choice of sign).



      Since the subgroups $langle irangle$,$langle jrangle$,$langle krangle$ are normal, none of these permutations is an inner automorphism. Thus they give a subgroup isomorphic to $S_3$ inside $Aut(Q_8)$, which trivially intersects $Inn(Q_8)cong Q_8/Z(Q_8)cong C_2times C_2$. Thus $Aut(Q_8)$ has size at least $|S_3|cdot |C_2times C_2|=24$ automorphisms.



      Since $Inn(Q_8)lhd Aut(Q_8)$, we get a semidirect product $(C_2times C_2)rtimes S_3$ inside $Aut(Q_8)$, which is then the whole group $Aut(Q_8)$. This is easily seen to be isomorphic to $S_4$, and so we are done.






      share|cite|improve this answer









      $endgroup$













      • $begingroup$
        "Since $i^2=j^2=k^2=-1$ is central and the only element of order $2$, we only need show $ alpha(i)alpha(j)=alpha(k);$ This is not always true, but what is always true is that $ alpha(i)alpha(j)=pmalpha(k),$ and so we see each permutation gives rise to an automorphism (after a suitable choice of sign)." I think the explanations are a bit too terse. Could you explain in more detail?
        $endgroup$
        – Makoto Kato
        Sep 15 '12 at 13:55














      5












      5








      5





      $begingroup$

      OK, let's first put an upper bound on the number of automorphisms of $Q_8$.



      There are $6$ elements of order $4$ in $Q_8$. It is well-known that $i$ and $j$ generate $Q_8$, so their images under some automorphism $phi$ are enough to determine that automorphism uniquely. So there are $6$ choices for $phi(i)$. Now we cannot have $phi(j)inlanglephi(i)rangle$, because then $phi$ would fail to be surjective. Thus there are $6-2=4$ possibilities for $phi(j)$. This crude reasoning gives the upper bound of $6cdot4=24$ automorphisms.



      Let $alpha$ be any permutation on the elements $lbrace i,j,krbrace$ (as a set). We can "extend" $alpha$ to a map of $Q_8$ in the obvious way (let it commute with negatives). Since any two of $lbrace i,j,krbrace$ generates $Q_8$, this map will be surjective, and since $Q_8$ is finite, injective as well. It only remains to show it is a homomorphism. Since $i^2=j^2=k^2=-1$ is central and the only element of order $2$, we only need show
      $$ alpha(i)alpha(j)=alpha(k);$$



      This is not always true, but what is always true is that
      $$ alpha(i)alpha(j)=pmalpha(k),$$
      and so we see each permutation gives rise to an automorphism (after a suitable choice of sign).



      Since the subgroups $langle irangle$,$langle jrangle$,$langle krangle$ are normal, none of these permutations is an inner automorphism. Thus they give a subgroup isomorphic to $S_3$ inside $Aut(Q_8)$, which trivially intersects $Inn(Q_8)cong Q_8/Z(Q_8)cong C_2times C_2$. Thus $Aut(Q_8)$ has size at least $|S_3|cdot |C_2times C_2|=24$ automorphisms.



      Since $Inn(Q_8)lhd Aut(Q_8)$, we get a semidirect product $(C_2times C_2)rtimes S_3$ inside $Aut(Q_8)$, which is then the whole group $Aut(Q_8)$. This is easily seen to be isomorphic to $S_4$, and so we are done.






      share|cite|improve this answer









      $endgroup$



      OK, let's first put an upper bound on the number of automorphisms of $Q_8$.



      There are $6$ elements of order $4$ in $Q_8$. It is well-known that $i$ and $j$ generate $Q_8$, so their images under some automorphism $phi$ are enough to determine that automorphism uniquely. So there are $6$ choices for $phi(i)$. Now we cannot have $phi(j)inlanglephi(i)rangle$, because then $phi$ would fail to be surjective. Thus there are $6-2=4$ possibilities for $phi(j)$. This crude reasoning gives the upper bound of $6cdot4=24$ automorphisms.



      Let $alpha$ be any permutation on the elements $lbrace i,j,krbrace$ (as a set). We can "extend" $alpha$ to a map of $Q_8$ in the obvious way (let it commute with negatives). Since any two of $lbrace i,j,krbrace$ generates $Q_8$, this map will be surjective, and since $Q_8$ is finite, injective as well. It only remains to show it is a homomorphism. Since $i^2=j^2=k^2=-1$ is central and the only element of order $2$, we only need show
      $$ alpha(i)alpha(j)=alpha(k);$$



      This is not always true, but what is always true is that
      $$ alpha(i)alpha(j)=pmalpha(k),$$
      and so we see each permutation gives rise to an automorphism (after a suitable choice of sign).



      Since the subgroups $langle irangle$,$langle jrangle$,$langle krangle$ are normal, none of these permutations is an inner automorphism. Thus they give a subgroup isomorphic to $S_3$ inside $Aut(Q_8)$, which trivially intersects $Inn(Q_8)cong Q_8/Z(Q_8)cong C_2times C_2$. Thus $Aut(Q_8)$ has size at least $|S_3|cdot |C_2times C_2|=24$ automorphisms.



      Since $Inn(Q_8)lhd Aut(Q_8)$, we get a semidirect product $(C_2times C_2)rtimes S_3$ inside $Aut(Q_8)$, which is then the whole group $Aut(Q_8)$. This is easily seen to be isomorphic to $S_4$, and so we are done.







      share|cite|improve this answer












      share|cite|improve this answer



      share|cite|improve this answer










      answered Sep 15 '12 at 2:39







      user641



















      • $begingroup$
        "Since $i^2=j^2=k^2=-1$ is central and the only element of order $2$, we only need show $ alpha(i)alpha(j)=alpha(k);$ This is not always true, but what is always true is that $ alpha(i)alpha(j)=pmalpha(k),$ and so we see each permutation gives rise to an automorphism (after a suitable choice of sign)." I think the explanations are a bit too terse. Could you explain in more detail?
        $endgroup$
        – Makoto Kato
        Sep 15 '12 at 13:55


















      • $begingroup$
        "Since $i^2=j^2=k^2=-1$ is central and the only element of order $2$, we only need show $ alpha(i)alpha(j)=alpha(k);$ This is not always true, but what is always true is that $ alpha(i)alpha(j)=pmalpha(k),$ and so we see each permutation gives rise to an automorphism (after a suitable choice of sign)." I think the explanations are a bit too terse. Could you explain in more detail?
        $endgroup$
        – Makoto Kato
        Sep 15 '12 at 13:55
















      $begingroup$
      "Since $i^2=j^2=k^2=-1$ is central and the only element of order $2$, we only need show $ alpha(i)alpha(j)=alpha(k);$ This is not always true, but what is always true is that $ alpha(i)alpha(j)=pmalpha(k),$ and so we see each permutation gives rise to an automorphism (after a suitable choice of sign)." I think the explanations are a bit too terse. Could you explain in more detail?
      $endgroup$
      – Makoto Kato
      Sep 15 '12 at 13:55




      $begingroup$
      "Since $i^2=j^2=k^2=-1$ is central and the only element of order $2$, we only need show $ alpha(i)alpha(j)=alpha(k);$ This is not always true, but what is always true is that $ alpha(i)alpha(j)=pmalpha(k),$ and so we see each permutation gives rise to an automorphism (after a suitable choice of sign)." I think the explanations are a bit too terse. Could you explain in more detail?
      $endgroup$
      – Makoto Kato
      Sep 15 '12 at 13:55











      3












      $begingroup$

      Hint:



      $Inn(Q_8)cong V$ and it equals its own centralizer in $Aut(Q_8)$. Now use $N/C$ Lemma in which $G=Aut(Q_8)$ and $H=Inn(Q_8)$. Of course using the lemma we always have $Inn(G)vartriangleleft Aut(G)$ and $G/Z(G)cong Inn(G)$ in which $G$ is our group.






      share|cite|improve this answer









      $endgroup$













      • $begingroup$
        What is $N/C$ Lemma?
        $endgroup$
        – Makoto Kato
        Sep 15 '12 at 13:58






      • 1




        $begingroup$
        If $Hleq G$, then $C_{G}(H)vartriangleleft N_{G}(H)$ and moreover $N_{G}(H)/C_{G}(H)hookrightarrow Aut(H)$. :)
        $endgroup$
        – mrs
        Sep 15 '12 at 14:50








      • 1




        $begingroup$
        math.stackexchange.com/a/30379/583 has the proof for Inn being its own centralizer in this case.
        $endgroup$
        – Jack Schmidt
        Sep 15 '12 at 18:45










      • $begingroup$
        $quad +1quad ddotsmilequad$
        $endgroup$
        – Namaste
        Mar 18 '13 at 0:56
















      3












      $begingroup$

      Hint:



      $Inn(Q_8)cong V$ and it equals its own centralizer in $Aut(Q_8)$. Now use $N/C$ Lemma in which $G=Aut(Q_8)$ and $H=Inn(Q_8)$. Of course using the lemma we always have $Inn(G)vartriangleleft Aut(G)$ and $G/Z(G)cong Inn(G)$ in which $G$ is our group.






      share|cite|improve this answer









      $endgroup$













      • $begingroup$
        What is $N/C$ Lemma?
        $endgroup$
        – Makoto Kato
        Sep 15 '12 at 13:58






      • 1




        $begingroup$
        If $Hleq G$, then $C_{G}(H)vartriangleleft N_{G}(H)$ and moreover $N_{G}(H)/C_{G}(H)hookrightarrow Aut(H)$. :)
        $endgroup$
        – mrs
        Sep 15 '12 at 14:50








      • 1




        $begingroup$
        math.stackexchange.com/a/30379/583 has the proof for Inn being its own centralizer in this case.
        $endgroup$
        – Jack Schmidt
        Sep 15 '12 at 18:45










      • $begingroup$
        $quad +1quad ddotsmilequad$
        $endgroup$
        – Namaste
        Mar 18 '13 at 0:56














      3












      3








      3





      $begingroup$

      Hint:



      $Inn(Q_8)cong V$ and it equals its own centralizer in $Aut(Q_8)$. Now use $N/C$ Lemma in which $G=Aut(Q_8)$ and $H=Inn(Q_8)$. Of course using the lemma we always have $Inn(G)vartriangleleft Aut(G)$ and $G/Z(G)cong Inn(G)$ in which $G$ is our group.






      share|cite|improve this answer









      $endgroup$



      Hint:



      $Inn(Q_8)cong V$ and it equals its own centralizer in $Aut(Q_8)$. Now use $N/C$ Lemma in which $G=Aut(Q_8)$ and $H=Inn(Q_8)$. Of course using the lemma we always have $Inn(G)vartriangleleft Aut(G)$ and $G/Z(G)cong Inn(G)$ in which $G$ is our group.







      share|cite|improve this answer












      share|cite|improve this answer



      share|cite|improve this answer










      answered Sep 15 '12 at 0:57









      mrsmrs

      1




      1












      • $begingroup$
        What is $N/C$ Lemma?
        $endgroup$
        – Makoto Kato
        Sep 15 '12 at 13:58






      • 1




        $begingroup$
        If $Hleq G$, then $C_{G}(H)vartriangleleft N_{G}(H)$ and moreover $N_{G}(H)/C_{G}(H)hookrightarrow Aut(H)$. :)
        $endgroup$
        – mrs
        Sep 15 '12 at 14:50








      • 1




        $begingroup$
        math.stackexchange.com/a/30379/583 has the proof for Inn being its own centralizer in this case.
        $endgroup$
        – Jack Schmidt
        Sep 15 '12 at 18:45










      • $begingroup$
        $quad +1quad ddotsmilequad$
        $endgroup$
        – Namaste
        Mar 18 '13 at 0:56


















      • $begingroup$
        What is $N/C$ Lemma?
        $endgroup$
        – Makoto Kato
        Sep 15 '12 at 13:58






      • 1




        $begingroup$
        If $Hleq G$, then $C_{G}(H)vartriangleleft N_{G}(H)$ and moreover $N_{G}(H)/C_{G}(H)hookrightarrow Aut(H)$. :)
        $endgroup$
        – mrs
        Sep 15 '12 at 14:50








      • 1




        $begingroup$
        math.stackexchange.com/a/30379/583 has the proof for Inn being its own centralizer in this case.
        $endgroup$
        – Jack Schmidt
        Sep 15 '12 at 18:45










      • $begingroup$
        $quad +1quad ddotsmilequad$
        $endgroup$
        – Namaste
        Mar 18 '13 at 0:56
















      $begingroup$
      What is $N/C$ Lemma?
      $endgroup$
      – Makoto Kato
      Sep 15 '12 at 13:58




      $begingroup$
      What is $N/C$ Lemma?
      $endgroup$
      – Makoto Kato
      Sep 15 '12 at 13:58




      1




      1




      $begingroup$
      If $Hleq G$, then $C_{G}(H)vartriangleleft N_{G}(H)$ and moreover $N_{G}(H)/C_{G}(H)hookrightarrow Aut(H)$. :)
      $endgroup$
      – mrs
      Sep 15 '12 at 14:50






      $begingroup$
      If $Hleq G$, then $C_{G}(H)vartriangleleft N_{G}(H)$ and moreover $N_{G}(H)/C_{G}(H)hookrightarrow Aut(H)$. :)
      $endgroup$
      – mrs
      Sep 15 '12 at 14:50






      1




      1




      $begingroup$
      math.stackexchange.com/a/30379/583 has the proof for Inn being its own centralizer in this case.
      $endgroup$
      – Jack Schmidt
      Sep 15 '12 at 18:45




      $begingroup$
      math.stackexchange.com/a/30379/583 has the proof for Inn being its own centralizer in this case.
      $endgroup$
      – Jack Schmidt
      Sep 15 '12 at 18:45












      $begingroup$
      $quad +1quad ddotsmilequad$
      $endgroup$
      – Namaste
      Mar 18 '13 at 0:56




      $begingroup$
      $quad +1quad ddotsmilequad$
      $endgroup$
      – Namaste
      Mar 18 '13 at 0:56











      1












      $begingroup$

      You can find the proof here: automorphism of generalized quaternionic group



      See Proposition 1.1 (pg. 156). Your case can be obtained taking $m=2$. But they prove for $mgeq 2$.






      share|cite|improve this answer











      $endgroup$


















        1












        $begingroup$

        You can find the proof here: automorphism of generalized quaternionic group



        See Proposition 1.1 (pg. 156). Your case can be obtained taking $m=2$. But they prove for $mgeq 2$.






        share|cite|improve this answer











        $endgroup$
















          1












          1








          1





          $begingroup$

          You can find the proof here: automorphism of generalized quaternionic group



          See Proposition 1.1 (pg. 156). Your case can be obtained taking $m=2$. But they prove for $mgeq 2$.






          share|cite|improve this answer











          $endgroup$



          You can find the proof here: automorphism of generalized quaternionic group



          See Proposition 1.1 (pg. 156). Your case can be obtained taking $m=2$. But they prove for $mgeq 2$.







          share|cite|improve this answer














          share|cite|improve this answer



          share|cite|improve this answer








          edited Sep 15 '12 at 2:38









          t.b.

          62.8k7208287




          62.8k7208287










          answered Sep 15 '12 at 2:11









          SigurSigur

          4,50311736




          4,50311736























              0












              $begingroup$

              The simplest way to determine the automorphism group of $Q_{8}$ may be to realize that $$Q_{8} text{char} SL(2,3) unlhd GL(2,3)$$ So $Q_{8} unlhd GL(2,3)$. One can take $$begin{equation*} x =left( begin{array}{cc} 0 & 1 \ -1 & 0 end{array} right), y = left( begin{array}{cc} 1 & -1 \ -1 & -1 end{array} right) end{equation*}$$ as the two generators of $Q_{8}$.



              Let $G=GL(2,3)$ and $Q=Q_{8}$. Then $C_{G}(Q) = Z(G)$ and therefore $PGL(2,3)=G/Z(G)$ is isomorphic to a subgroup of $Aut(Q)$. Now $|G|=48$ and $|Z(G)|=2$. Thus $|G/Z(G)|=24$. As others have shown $|Aut(Q)| leq 24$. Hence we can conclude that $$PGL(2,3) simeq Aut(Q_{8})$$
              Using that $SL(2,3)$ and therefore $GL(2,3)$ have $4$ Sylow-$3$ subgroups one get a homomorphism into $Sym(4)$. It is straight forward to show that the kernel of this homomorphism is $Z(GL(2,3))$ and therefore $PGL(2,3)$ is isomorphic to a subgroup of $Sym(4)$. Since the two groups have the same number of elements we are done.






              share|cite|improve this answer









              $endgroup$


















                0












                $begingroup$

                The simplest way to determine the automorphism group of $Q_{8}$ may be to realize that $$Q_{8} text{char} SL(2,3) unlhd GL(2,3)$$ So $Q_{8} unlhd GL(2,3)$. One can take $$begin{equation*} x =left( begin{array}{cc} 0 & 1 \ -1 & 0 end{array} right), y = left( begin{array}{cc} 1 & -1 \ -1 & -1 end{array} right) end{equation*}$$ as the two generators of $Q_{8}$.



                Let $G=GL(2,3)$ and $Q=Q_{8}$. Then $C_{G}(Q) = Z(G)$ and therefore $PGL(2,3)=G/Z(G)$ is isomorphic to a subgroup of $Aut(Q)$. Now $|G|=48$ and $|Z(G)|=2$. Thus $|G/Z(G)|=24$. As others have shown $|Aut(Q)| leq 24$. Hence we can conclude that $$PGL(2,3) simeq Aut(Q_{8})$$
                Using that $SL(2,3)$ and therefore $GL(2,3)$ have $4$ Sylow-$3$ subgroups one get a homomorphism into $Sym(4)$. It is straight forward to show that the kernel of this homomorphism is $Z(GL(2,3))$ and therefore $PGL(2,3)$ is isomorphic to a subgroup of $Sym(4)$. Since the two groups have the same number of elements we are done.






                share|cite|improve this answer









                $endgroup$
















                  0












                  0








                  0





                  $begingroup$

                  The simplest way to determine the automorphism group of $Q_{8}$ may be to realize that $$Q_{8} text{char} SL(2,3) unlhd GL(2,3)$$ So $Q_{8} unlhd GL(2,3)$. One can take $$begin{equation*} x =left( begin{array}{cc} 0 & 1 \ -1 & 0 end{array} right), y = left( begin{array}{cc} 1 & -1 \ -1 & -1 end{array} right) end{equation*}$$ as the two generators of $Q_{8}$.



                  Let $G=GL(2,3)$ and $Q=Q_{8}$. Then $C_{G}(Q) = Z(G)$ and therefore $PGL(2,3)=G/Z(G)$ is isomorphic to a subgroup of $Aut(Q)$. Now $|G|=48$ and $|Z(G)|=2$. Thus $|G/Z(G)|=24$. As others have shown $|Aut(Q)| leq 24$. Hence we can conclude that $$PGL(2,3) simeq Aut(Q_{8})$$
                  Using that $SL(2,3)$ and therefore $GL(2,3)$ have $4$ Sylow-$3$ subgroups one get a homomorphism into $Sym(4)$. It is straight forward to show that the kernel of this homomorphism is $Z(GL(2,3))$ and therefore $PGL(2,3)$ is isomorphic to a subgroup of $Sym(4)$. Since the two groups have the same number of elements we are done.






                  share|cite|improve this answer









                  $endgroup$



                  The simplest way to determine the automorphism group of $Q_{8}$ may be to realize that $$Q_{8} text{char} SL(2,3) unlhd GL(2,3)$$ So $Q_{8} unlhd GL(2,3)$. One can take $$begin{equation*} x =left( begin{array}{cc} 0 & 1 \ -1 & 0 end{array} right), y = left( begin{array}{cc} 1 & -1 \ -1 & -1 end{array} right) end{equation*}$$ as the two generators of $Q_{8}$.



                  Let $G=GL(2,3)$ and $Q=Q_{8}$. Then $C_{G}(Q) = Z(G)$ and therefore $PGL(2,3)=G/Z(G)$ is isomorphic to a subgroup of $Aut(Q)$. Now $|G|=48$ and $|Z(G)|=2$. Thus $|G/Z(G)|=24$. As others have shown $|Aut(Q)| leq 24$. Hence we can conclude that $$PGL(2,3) simeq Aut(Q_{8})$$
                  Using that $SL(2,3)$ and therefore $GL(2,3)$ have $4$ Sylow-$3$ subgroups one get a homomorphism into $Sym(4)$. It is straight forward to show that the kernel of this homomorphism is $Z(GL(2,3))$ and therefore $PGL(2,3)$ is isomorphic to a subgroup of $Sym(4)$. Since the two groups have the same number of elements we are done.







                  share|cite|improve this answer












                  share|cite|improve this answer



                  share|cite|improve this answer










                  answered Dec 17 '18 at 20:31









                  user515430user515430

                  17613




                  17613






























                      draft saved

                      draft discarded




















































                      Thanks for contributing an answer to Mathematics Stack Exchange!


                      • Please be sure to answer the question. Provide details and share your research!

                      But avoid



                      • Asking for help, clarification, or responding to other answers.

                      • Making statements based on opinion; back them up with references or personal experience.


                      Use MathJax to format equations. MathJax reference.


                      To learn more, see our tips on writing great answers.




                      draft saved


                      draft discarded














                      StackExchange.ready(
                      function () {
                      StackExchange.openid.initPostLogin('.new-post-login', 'https%3a%2f%2fmath.stackexchange.com%2fquestions%2f195932%2fautomorphism-group-of-the-quaternion-group%23new-answer', 'question_page');
                      }
                      );

                      Post as a guest















                      Required, but never shown





















































                      Required, but never shown














                      Required, but never shown












                      Required, but never shown







                      Required, but never shown

































                      Required, but never shown














                      Required, but never shown












                      Required, but never shown







                      Required, but never shown







                      Popular posts from this blog

                      Plaza Victoria

                      In PowerPoint, is there a keyboard shortcut for bulleted / numbered list?

                      How to put 3 figures in Latex with 2 figures side by side and 1 below these side by side images but in...